Abtastung und Leckeffekt

This commit is contained in:
WieErWill 2022-03-17 22:48:18 +01:00
parent 2b1f8cd8b7
commit 180da445f0
2 changed files with 45 additions and 10 deletions

Binary file not shown.

View File

@ -388,13 +388,37 @@
\begin{solution}
Um Signalfrequenz $f_c$ mit einer der diskreten Frequenzen im DFT-Spektrum zu treffen, muss die Signalfrequenz $f_c$ ein ganzzahliges Vielfaches der spektralen Auflösung $\delta f$ sein: $f_c=\frac{1}{T_c=k*\Delta f=\frac{k}{T_{DFT}}}$. Die Analysezeit $T_{DFT}$ sollte ebenso ganzes vielfaches der Periodendauer $T_C$ sein: $T_{DFT}=k*T_C=N*T_A$
$T_M = \frac{1}{T_{DFT}}=1/4,5s = ...$
$fa= N\ Samples * T_M = ...$
$fc = N\ Perioden * T_M =...$
$fs=0$
$df=1/T_M$
$fe=fa-df=...$
\end{solution}
\part Wodurch wird hoher Spektralteil links im DFT Betragsspektrum verursacht?
\begin{solution}
Spektralwert bei $f_s$ entspricht dem Gleichanteil des Signals. Dieser ist im angegebenen Signal nicht Null sondern 0,5.
$y(t)=cos(2\pi f_c t)+ 0,5$
Dass dieses Spektrum nicht exakt 0,5 ist, liegt am Leckeffekt
\end{solution}
\part Welcher Effekt verursacht viele Spektralteile?
\begin{solution}
Leckeffekt: Signal wird in Blöcken verarbeitet, diese Blöcke sind endlich $\rightarrow$ Leckeffekt entsteht, wenn Blocklänge nicht natürlichzahliges Vielfaches der Periode des Signals ist.
Leckeffekt führt zu Verzerrungen und falsch detektierte Spektraleanteilen durch Überlagerung unterschiedlicher Perioden
\begin{itemize}
\item Leckeffekt = Auslaufen der Spektralanteile auf benachbarte Frequenzen $\Rightarrow$ führt zu Verzerrungen und falsch detektierte Spektraleanteilen durch Überlagerung unterschiedlicher Perioden
\item Zeitbereich: Periodifizierung der Abtastwerte ergibt kein cos-Signal sondern erzeugt Sprungstellen an Rändern
\item Frequenzbereich: ausschneiden des Signals entspricht im Spektralbereich einer Faltung mit Fourtiertransformierten dieses Rechteckfensters. Signal wird in Blöcken verarbeitet, diese Blöcke sind endlich $\rightarrow$ Leckeffekt entsteht, wenn Blocklänge nicht natürlichzahliges Vielfaches der Periode des Signals ist.
\end{itemize}
\end{solution}
\part Welche Eigenschaft muss eine Fensterfunktion haben damit dieser Effekt verringert wird?
@ -412,6 +436,22 @@
\item[breite Fensterung] steile Übergänge, geringe Sperrdämpfung
\end{description}
\end{solution}
\part Gebe Abtastfrequenz $f_A$ und Messdauer $T_M$ an, so dass das resultierende DFT-Spektrum exakt wird
\begin{solution}
$T_M$ muss natürliches vielfaches von $T_C=1/f_C$ sein und zugleich vielfaches von $T_A=1/f_A$ sein, wobei $f_A$ das Abtasttheorem einhalten muss.
$$T_M=M*T_C=M*\frac{1}{f_C}=N*T_A=N*\frac{1}{f_A}$$
Bsp:
$T_M=\frac{1}{7} s$, $f_A=21Hz$, $N=3\ Samples$, $M=1$
$T_M=\frac{7}{7} s$, $f_A=21Hz$, $N=21\ Samples$, $M=7$
$T_M=2 s$, $f_A=21Hz$, $N=42\ Samples$, $M=14$
\end{solution}
\end{parts}
\question Filter
@ -535,13 +575,8 @@
\end{solution}
\end{parts}
\question Berechne mit $t_{ab}=10s$, $f_{s1}=9kHz$ und $f_{s2}=10kHz$, peaks gegeben
\question Berechne mit $t_{ab}=10s$, $f_{s1}=9kHz$ und $f_{s2}=10kHz$, Peaks gegeben
\begin{parts}
\part Abtasttheorem
\begin{solution}
\end{solution}
\part Welche Frequenzbereiche der Signale
\begin{solution}